Sunteți pe pagina 1din 65

T

Exercise 1:
Suppose that as a result of an improvement in technology
the producer's supply change from:
(S1) Qs=-40+20P to (S2) Qs=-10+20P
1. From S1 to S2, Supply increase or decrease? Why?
2. Derive this producer's old and new supply schedule?
3. On one axes, draw this producer's supply curves before
and after the improvement in technology?
4. How much of commodity X does this producer supply at
the price of 4$ before and after the improvement in technology?
Q(s2)-Q(s1)=30 => Supply increased
P
2
3
4
Q(S1)
0
20
40
Q(S2)
30
50
70

5
60
90

6
4

P
2
0
0

10

20

30

P=4, Q(S1)=40, Q(S2)=70

40

S150

S260

70

80

90

100

Exercise 2:
Tablele 2.1 gives the supply schedule of the three producer of
commodity X in the market. Draw, on one set of axes,
the three producer's supply curves and derice geometrically
the market supply curve for commodity X
P
($/Kg)
Quantity Supplied (Kg, Per time period)
Producer 1
Producer 2
Producer 3 Market Supply
0
0
0
10
10
1
0
0
25
25
2
0
20
35
55
3
10
30
42
82
4
16
36
46
98
5
20
40
50
110
6
22
42
53
117

X
7
6
5
4

D1

D2

D3
S

1
0
0

20

40

60

80

100

120

Exercise 3:
There are 10,000 identical individual in the market for
commodity X, each with a demand function given by
(D1) Qd=12-2P, and 1,000 identical producers of commodity X,
each with a function given by (S1) Qs=20P.
1. Find the market demand function and the market supply
function for commodity X?
2. Find the market demand schedule and the market supply
schedule of commodity X and from them find the equilibrium
price and the equilibrium quantity?
3. Plot, on one set of axes, the market demand curve and the
market supply curve for commodity X and show the equilibrium
point?
4. Obtain the equilibrium price and the equilibrium quantity mathematically?
S1->S2 increase x%
(S1) P=a+bQ -> S2 P=a+b/(1+x%)Q
S1 -> S2 increase x%
(S1) Q=c+dP=>Q=(1+x%)(c+dP)
Ex1: Qd=120000-20000P; Qs=20000P
P
Qd
Qs
1
100,000
20,000
2
80,000
40,000
3
60,000
60,000
4
40,000
80,000
5
20,000
100,000
6
0
120,000

X
7
6
5

4
3

2
1
0
0

50,000

100,000

150,000

Q
D

Qd=Qs<=> 120000-20000P=20000P <=> P = 3 ; Q=60000

Exercise 4:
Suppose that from the condition of equilibrium in exercise 3, there is an
increase in consumer's income (ceteris paribus) so that a new market demand
curve is given by (D2) Qd=140,000-20,000P
1. Derive the new market demand schedule
2. Show the new market demand curve on the graph of exercise 3 (3)
3.State the new equilibrium price and the new equilibrium quantity for
commodity X.
4. Obtain the equilibrium price and the equilibrium quantity mathematically?
P
0
0.5
1.0
1.5
2.0
2.5
3.0
3.5
4.0
4.5
5.0

Qs
0
10,000
20,000
30,000
40,000
50,000
60,000
70,000
80,000
90,000
100,000

Qd
140,000
130,000
120,000
110,000
100,000
90,000
80,000
70,000
60,000
50,000
40,000

X
6
5
4

3
2
1
0
0

50,000

100,000
S

150,000

Exercise 5:
Commodity X have:
(D1) Q=100-2P
(S1) Q=3P-50
1. Find the equilibrium price Pe1and the equilibrium quantity Qe1?
Pe1=
50.00
Qe1=
0.00
2 Demand increase 50% become (D2), so demand D2 have:
(D2) 1.5Q(D1)=1.5(100-2P)=150-3P
Find the equilibrium price Pe2 and the equilibrium quantity Qe2 between D2 and S1?
Pe2=
33.33
Qe2=
50.00
3 Supply increase 40% become (S2), so supply S2 have:
(S2) Q=1,4Q(S1)=1,4(3P-50)=4,2P-70
Find the equilibrium price Pe3 and the equilibrium quantity Qe3 between D2 and S2?
Pe3=
30.56
Qe3=
58.33
4. Derive the demand schedule and supply schedule
Q
Pd1
Ps1
Pd2
Ps2
0.00
50.00
16.67
50.00
16.67
5.00
47.50
18.33
48.33
17.86
.. .. ..
..
..
.
.
.
.
.
75.00
12.50
41.67
25.00
34.52
80.00
10.00
43.33
23.33
35.71
Q
0.00
5.00
10.00
15.00
20.00
25.00
30.00
35.00
40.00
45.00
50.00
55.00
60.00
65.00
70.00
75.00
80.00
85.00

Pd1
50.00
47.50
45.00
42.50
40.00
37.50
35.00
32.50
30.00
27.50
25.00
22.50
20.00
17.50
15.00
12.50
10.00
7.50

Ps1
16.67
18.33
20.00
21.67
23.33
25.00
26.67
28.33
30.00
31.67
33.33
35.00
36.67
38.33
40.00
41.67
43.33
45.00

Pd2
50.00
48.33
46.67
45.00
43.33
41.67
40.00
38.33
36.67
35.00
33.33
31.67
30.00
28.33
26.67
25.00
23.33
21.67

Ps2
16.67
17.87
19.07
20.27
21.47
22.67
23.87
25.07
26.27
27.47
28.67
29.87
31.07
32.27
33.47
34.67
35.87
37.07

5. Plot, on one set of axes, the demand curve (D1, D2) and the
supply curve (S1, S2) for commodity X and show the equilibrium
point (E1, E2, and E3)?

X
60.00
50.00
40.00
30.00

E1

E3
E2

E4

20.00
10.00
0.00
0.00

10.00

20.00

30.00
D1

40.00
S1

50.00
D2

60.00
S2

70.00

80.00

90.00

Exercise 6:
Commodity X have:
(D1) P=100-(1/4)Q
(S1) P=(3/4)Q-50
1. Find the equilibrium price, Pe1and the equilibrium quantity, Qe1?
Qe1=150
Pe1=62,5
2 Demand increase 50% become (D2), so demand D2 have:
(D1) Q=400-4P => (D2) Q= (1+0,5)(400-4P)=600-6P
Find the equilibrium price Pe2 and the equilibrium quantity Qe2 between D2 and S1?
Pe2=800/11
Qe2=1800/11
3 Supply increase 40% become (S2), so supply S2 have:
(S1) Q=4P/3+200/3 => (S2) Q=(1+0,4)(4P/3+200/3) <=> Q=28/15P+280/3
Find the equilibrium price Pe3 and the equilibrium quantity Qe3 between D2 and S2?
4. Derive the demand schedule and supply schedule
5. Plot, on one set of axes, the demand curve (D1, D2) and the
supply curve (S1, S2) for commodity X and show the equilibrium
point (E1, E2, and E3)?
P
Qs1
Qd1
Qs2
Qd2
45.00
126.67
220.00
177.33
330.00
52.27
136.36
190.91
190.91
286.36
61.00
148.00
156.00
207.20
234.00
62.50
150.00
150.00
210.00
225.00
64.00
152.00
144.00
212.80
216.00
64.41
152.54
142.37
213.56
213.56
72.73
163.64
109.09
229.09
163.64
75.00
166.67
100.00
233.33
150.00
80.00
173.33
80.00
242.67
120.00

X
85.00
80.00
75.00
70.00
65.00
60.00
55.00
50.00
45.00
40.00
50.00

E2
E3

E1
E4

100.00

150.00
S1

200.00
D1

250.00
S2

D2

300.00

350.00

Exercise 7:
There are 10,000 identical individual in the market for
commodity X, each with a demand function given by
(D1) P=120-Q, and 1,000 identical producers of commodity X,
each with a function given by (S1) P=(1/2)Q
1. Find the market demand function and the market supply
function for commodity X?
Pd= 120-1/10000Q
Ps= 1/2000Q
2. Find the market demand schedule and the market supply
schedule of commodity X and from them find the equilibrium
price and the equilibrium quantity?
P
98
99
100
101
102
103

Qs
196,000
198,000
200,000
202,000
204,000
206,000

Qd
220,000
210,000
200,000
190,000
180,000
170,000

X
104
102
100
98
96
160,000

180,000

200,000
S

220,000

3. Plot, on one set of axes, the market demand curve and the
market supply curve for commodity X and show the equilibrium
point?
4. Obtain the equilibrium price and the equilibrium quantity mathematically?
Equilibrium quantity: Ps=Pd <=> Q=100

Exercise 1:
The following table presents hypothetical data for the market demand
for a good. Complete the table:
Qd
P
AR
TR
MR
1.00
50.00
50.00
50.00
40.00
2.00
40.00
40.00
80.00
20.00
3.00
30.00
30.00
90.00
0.00
4.00
20.00
20.00
80.00
-20.00
5.00
10.00
10.00
50.00
-40.00
6.00
0.00
0.00
0.00
-60.00
AR=TR/Q=P=Doanh thu trung bnh=Average Revenue
MR=TR/Q==dTR/dQ=Doanh thu bin=Marginal Revenue
Ed=Ep=Edp=(%Q/%P)=(Q/P)*(P/Q)=[1/((P/Q)]*(P/Q)
Ed=Ep=Edp=(%Q/%P)=(dQ/dP)*(P/Q)=[1/(dP/dQ)]*(P/Q)
Ed= -1<=>Unitary Elastic
P=
60.00 +
Ed< -1<=>Elastic (Co gin nhiu)
P=60-10Q
Ed> -1<=>Inelastic (Co gin t)
TR=PQ=60Q-10Q^2
Ed= 0<=>Perfectly Inelastic (HT kg co gin)
MR=TR'=60-20Q
Ed= -<=>Perfectly Elastic (HT co gin)
Ed=1/-10*P/Q

Ed
-5.00
-2.00
-1.00
-0.50
-0.20
0.00

Type of Demand
Elastic
Elastic
Unitary Elastc
Inelastic
Inelastic
Inelastic

-10.00 *Q

Exercise 2:
Given: The demand equation is P=40-2Q
Q=20-P/2
TR=PQ=40Q-2Q^2
a. What is the equation for MR?
MR=(TR)'=40-4Q
b. At what output is MR=0?
MR=0<=>40-4Q=0<=>Q=10
c. At what output is TR maximum?
TRmax<=>Q=-40/2(-2)=10
d. Determine the price elasticity of demand at the output where TR is maximum
Complete the table:
Q
P
TR
MR
Ed
Type of Demand
0.00
40.00
0.00
40.00

Perfect Elastic
0.50
39.00
19.50
38.00
-39.00
Elastic
1.00
38.00
38.00
36.00
-19.00
Elastic
1.50
37.00
55.50
34.00
-12.33
Elastic
2.00
36.00
72.00
32.00
-9.00
Elastic
2.50
35.00
87.50
30.00
-7.00
Elastic
3.00
34.00
102.00
28.00
-5.67
Elastic
3.50
33.00
115.50
26.00
-4.71
Elastic
4.00
32.00
128.00
24.00
-4.00
Elastic
4.50
31.00
139.50
22.00
-3.44
Elastic
5.00
30.00
150.00
20.00
-3.00
Elastic
5.50
29.00
159.50
18.00
-2.64
Elastic
6.00
28.00
168.00
16.00
-2.33
Elastic
6.50
27.00
175.50
14.00
-2.08
Elastic
7.00
26.00
182.00
12.00
-1.86
Elastic
7.50
25.00
187.50
10.00
-1.67
Elastic
8.00
24.00
192.00
8.00
-1.50
Elastic
8.50
23.00
195.50
6.00
-1.35
Elastic
9.00
22.00
198.00
4.00
-1.22
Elastic
9.50
21.00
199.50
2.00
-1.11
Elastic
10.00
20.00
200.00
0.00
-1.00
Inelastic
10.50
19.00
199.50
-2.00
-0.90
Inelastic
11.00
18.00
198.00
-4.00
-0.82
Inelastic
11.50
17.00
195.50
-6.00
-0.74
Inelastic
12.00
16.00
192.00
-8.00
-0.67
Inelastic

12.50
15.00
187.50
13.00
14.00
182.00
13.50
13.00
175.50
14.00
12.00
168.00
14.50
11.00
159.50
15.00
10.00
150.00
15.50
9.00
139.50
16.00
8.00
128.00
16.50
7.00
115.50
17.00
6.00
102.00
17.50
5.00
87.50
18.00
4.00
72.00
18.50
3.00
55.50
19.00
2.00
38.00
19.50
1.00
19.50
20.00
0.00
0.00
Draw, on one set of axes the P, TR, MR

Chart Title

-10.00
-12.00
-14.00
-16.00
-18.00
-20.00
-22.00
-24.00
-26.00
-28.00
-30.00
-32.00
-34.00
-36.00
-38.00
-40.00

-0.60
-0.54
-0.48
-0.43
-0.38
-0.33
-0.29
-0.25
-0.21
-0.18
-0.14
-0.11
-0.08
-0.05
-0.03
0.00

Inelastic
Inelastic
Inelastic
Inelastic
Inelastic
Inelastic
Inelastic
Inelastic
Inelastic
Inelastic
Inelastic
Inelastic
Inelastic
Inelastic
Inelastic
Perfectly Inelastic

160
160.00
110
110.00
60
60.00
10
10.00
0
-40 0.00
-40.00

2.00

4.00

6.00

10

8.00
P

15
10.00

12.00

TR

MR

20
14.00

16.00

18.00

20.00

Exercise 3:
Suppose that the demand equation for a good is Q=20-2P
Complete the table:
Q
P
TR
MR
0.00
2.00
4.00

20.00
TR=PQ=20P-2P^2
MR=TR'=20-4P
Q
P
TR
MR
0.00
10.00
0.00
-20.00
2.00
9.00
18.00
-16.00
4.00
8.00
32.00
-12.00
6.00
7.00
42.00
-8.00
8.00
6.00
48.00
-4.00
10.00
5.00
50.00
0.00
12.00
4.00
48.00
4.00
14.00
3.00
42.00
8.00
16.00
2.00
32.00
12.00
18.00
1.00
18.00
16.00
20.00
0.00
0.00
20.00

Ed

Ed

-2.25
-1.00
-0.58
-0.38
-0.25
-0.17
-0.11
-0.06
-0.03
0.00

Type of Demand

Type of Demand
Perfect Elastic
Elastic
Inelastic
Inelastic
Inelastic
Inelastic
Inelastic
Inelastic
Inelastic
Inelastic
Perfectly Inelastic

X
250.00
200.00
150.00

100.00
50.00
0.00
-50.00

0.00

5.00

10.00

-100.00

15.00

Q
TR

MR

20.00

25.00

Exercise 4:
2
Suppose that the demand equation for a good is Q=16+9P-2P ,
calculate the price elasticity of demand at a price of $4 and at a price $3
Ed=dQ/dP*P/Q=(9-4P)*P/Q
Q
P
Ed
20.00
4.00
1.80
25.00
3.00
1.08
Exercise 5:
2
If the demand equation for an item is P=1000+3Q-4Q
a. Determine price elasticity of demand at Q=10
b. Determine the equation for TR and MR
Ed=1/(3-8Q)*P/Q
TR=PQ=1000Q+3Q^2-4Q^3
MR=TR'=1000+6Q-12Q^2
Q
P
Ed
10.00
630.00
21.00
Exercise 6:
2
Given: The relationship between product A and product B is Qa=80Pb-0.5Pb ,
where Qa=Units of product A demanded by consumers each day and Pb=Selling
price of product B.
a. Determine the cross-elasticity coefficient for the two products when the price
of product B=$10
b. Are products A and B complements, subtitutes, or independent, and how "strong"
is the relationship?
Qa
Pb
Ed
750.00
10.00
0.93
The relationship is weak because Ed>-1
A and B are subtitues

Exercise 7:
The Fairfax Apparel Company manufactures sports, shirts for men; during 1987 Qa1=
Fairfax sold an average of 23,000 sports shirts for $13 per shirt. In early January Pa=
1988, Fairfax's major competitor, Lafayyete Manufacturing Co., cut the price of Pb1=
its sports shirts from $15 to $12. The orders Fairfax received for its own sports Pb2=
shirts dropped sharly, from 23,000 per month to 13,000 per month for February Qa2=
and March 1988.
a. Calculate the cross elasticity of demand between Fairfax's sports shirts and
Lafayette's sports shirts during February and March. Are the two companies'
sports shirts good or poor substitutes?
%Pb=
-20.00%
%Qa=
-43.48%
%Pb(tb)=
%Qa(tb)=
-22.22%
-55.56% =Q/Qtb=(Q2-Q1)/[(Q1+Q2)/2]
Eab=
2.17
Eab(tb)=
2.50
b. Suppose that the coeffient of the price elasticity of demand for Fairfax's
sports shirts is -2.0. Assuming that Lafayette keeps its price at $12, by how
much must Fairfax cut its price to build its sales of shirts back up to 23,000
per month? (Use the arc formula for price elasticity)
Ed=-2
Pb=12
Qa=23000
Pa=?
Eab(tb)=%Qa(tb)/%Pb(tb)=[(Qa3-Qa2)/(Qa3+Qa2)]/[(Pb3-Pb2)/(Pb3+Pb2)]
36.14

23,000.00
13.00
15.00
12.00
13,000.00

Exercise 8:
Find the price elasticity of demand (Ed) for the curvilinear demand function
of the form Q=aP-b
Exercise 9:
Suppose that two prices and their corresponding quantities (Table 9) are
observed in the market for commodity X. Find the price elasticity of demand
for commodity X between point A and point B (Moving from A to B, from
B to A, and Midway between A and B)
Point
Px
Qx
A
6.10
32,180.00
B
5.70
41,230.00
A->B
%P=
-0.07
%Q=
0.28
B->A
%P=
0.07
%Q=
-0.22
Midpoint
%P=
-0.07
%Q=
0.25
Exercise 10:
Find the cross elasticity of demand between hot dogs (X) and hamburgers (Y) (Exy)
and between hot dogs (X) and mustard (Z) (Exz) for the data in Table 10
Commodity
Hamburgers (Y)
Hot dogs (X)
Mustard (Z)
Hot dogs (X)
Exy=
Exz=

Before
P
3.00
1.00
1.50
1.00
1.00
-0.78

After
Q
30.00
15.00
10.00
15.00

P
2.00
1.00
2.00
1.00

Q
40.00
10.00
9.00
12.00

%P

%Q
-0.40
0.00
0.29
0.00

0.29
-0.40
-0.11
-0.22

Exercise 11:
From the supply schedule in Table 11, find arc elasticity for a movement
a. From poit A to point C
b. From poit C to point A
c. Midway between A and C
d. At point B
Point
A
B
C
D
Px
6.00
5.00
4.00
3.00
Qx
6,000.00
5,500.00
4,500.00
3,000.00
Midway AC
Movement
A->C
C->A
A->C
C->A
%P
-33.33%
50.00%
-40.00%
40.00%
%Q
-25.00%
33.33%
-28.57%
28.57%
Ed
0.75
0.67
0.71
0.71

E
2.00
0.00
At point B
B->A
20.00%
9.09%
0.45

B->C
-20.00%
-18.18%
0.91

Exercise 12:
With reference to Fig 12, consider the following two farm-aid programs for
wheat farmers.
I. The government sets the price of wheat at P2 and purchases the resulting
surplus of wheat at P2.
II. The government allows wheat to be sold at the equilibrium price of P1
and grants each farmer a cash subsidy of P2-P1 on each unit sold. Which
of the two programs is more expensive to the government?
Sw

Pw
P2
P1

A
E
Dw

Qw

Depending on elasticity of the demand function


Elastic
I>II
Inelastic
I<II
Unitary Elastic I=II

Exercise 13:
We have:
%Q=
%TR=
Find:
%P=
Ed=
Exercise 14:
We have:
%P=
%TR=
Find:
%Q=
Ed=
Exercise 15:
We have:
%Q=
%P=
Find:
%TR=
Ed=
Exercise 16:
We have:
%Q=
Ed=
Find:
%P=
%TR=

10.00%
6.00%
-3.64%
-2.75

12.00%
8.00%
-3.57%
-0.30

20.00%
-4.00%
15.20%
-5.00

8.00%
-4.00
-2.00%
5.84%

Exercise 17:
At the equilibrium point we have:
Pe=
100.00
Qe=
200.00
Es=
4.00
Ed=
-2.00
Find:
Demand function and Supply function:
Q=a+bP and P=c+dQ
Ed=dQ/dP*P/Q <=> dQ/dP * 100/200=-2 <=> dQ/dP=-4 => b=04
(D)
Q
=
600.00
Es=1/(dP/dQ) * P/Q <=> 1/(dP/dQ)*100/200=4 <=> 1/(dP/dQ)=8 <=> d=1/8
(S)
P
=
75.00
+

4.00

1/8

Exercise 18:
A
B

Compare type of demand at poits A, B, and C


1.00
Assume that:
D1 Q=
9.00 +
D2 Q=
11.00 +
Q
P1
P2
1.00
8.00
10.00
2.00
7.00
9.00
3.00
6.00
8.00
4.00
5.00
7.00
5.00
4.00
6.00
6.00
3.00
5.00
7.00
2.00
4.00
8.00
1.00
3.00
Q
P
A
3.00
8.00
B
3.00
6.00
C
5.00
6.00

-1.00 *P
-1.00 *P
Ed1
Ed2
-8.00
-10.00
-7.00
-4.50
-4.00
-2.67
-2.50
-1.75
-1.60
-1.20
-1.00
-0.83
-0.57
-0.57
-0.25
-0.38
Ed
Type of demand
-2.67 Elastic
-2.00 Elastic
-1.20 Elastic

X
12.00
10.00
8.00
P

6.00
P1
4.00

P2

2.00
0.00
0.00

2.00

4.00

6.00
Q

8.00

10.00

Exercise 19:
A
B

Compare type of demand at poits A, B, and C


5.00
1.00
1.00
Assume that:
D1 Q=
-5.00 +
D2 Q=
-1.00 +
Q
P1
P2
-30.00
5.00
29.00
-20.00
3.00
19.00
-10.00
1.00
9.00
1.00
-1.20
-2.00
10.00
-3.00
-11.00
20.00
-5.00
-21.00
30.00
-7.00
-31.00
50.00
-11.00
-51.00
Q
P
A
10.00
-3.00
B
10.00
-11.00
C
50.00
-11.00

-5.00 *P
-1.00 *P
Ed1

Ed2
0.83
0.30
0.20
2.40
0.60
0.50
0.47
0.44

0.97
0.95
0.90
2.00
1.10
1.05
1.03
1.02
Ed
Type of demand
0.30 Inelastic
1.10 Inelastic
0.22 Inelastic

P -40.00

-20.00

40.00
30.00
20.00
10.00
0.00
-10.00 0.00
-20.00
-30.00
-40.00
-50.00
-60.00

20.00

40.00

60.00

P1
P2

Exercise 20:
A
B

Compare type of demand at poits A, B, and C


1.00
4.00
10.00
Assume that:
D1 Q=
10.00 +
D2 Q=
10.00 +
Q
P1
P2
1.00
9.00
2.25
3.00
7.00
1.75
5.00
5.00
1.25
7.00
3.00
0.75
9.00
1.00
0.25
11.00
-1.00
-0.25
13.00
-3.00
-0.75
15.00
-5.00
-1.25
Q
P
A
10.00
9.00
B
10.00
2.25
C
3.00
7.00

-1.00 *P
-4.00 *P
Ed1
Ed2
-9.00
-9.00
-2.33
-2.33
-1.00
-1.00
-0.43
-0.43
-0.11
-0.11
0.09
0.09
0.23
0.23
0.33
0.33
Ed
Type of demand
-0.90 Inelastic
-0.23 Inelastic
-2.33 Elastic

X
10.00
8.00
6.00
4.00
P

2.00

P1

0.00

P2

-2.00 0.00

2.00

4.00

6.00

8.00

-4.00
-6.00

10.00 12.00 14.00 16.00

Exercise 1:
Demand and Supply for commodity X:
(D1) Q=500-2P
(S1) Q=3P-200
1. Find the equilibrium price and quantity (Pe1, Qe1) for commodity X,
elasticity of demand and supply at equilibrium point (Ed1, Es1)?
2. Demand for commodity X increase 50% in order to become D2,
(D2) Q=???
Pe1=
140.00
Ed1=
-1.27
Qe1=
220.00
Es1=
1.91
(D2)
Q=1.5*(500-2P)=750-3P
Find the equilibrium price and quantity (Pe2, Qe2) between D2 and S1
Pe2=
158.33
Ed2=
-1.73
Qe2=
275.00
Es2=
1.73
3. Supply for commodity X increase 40% in order to become S2,
(S2) Q=1.4*(3P-200)=4.2P-280
Find the equilibrium price and quantity (Pe3, Qe3) between D2 and S2
Pe3=
143.06
Ed3=
-1.34
Qe3=
320.83
Es3=
1.87
4. From D2 and S2, the government set the price control (Pct)
4.1 Price control=120%*Pe3
Pct=???
171.67
What is type of price?
Floor price
Result of that price:
Qd=1.5*(500-2P)=750-3P
Qs=1.4*(3P-200)=4.2P-280
Qd=???
235.00
Qs=???
441.00
Excess demand or excess supply?
Excess supply
206.00

Government spending (B) ???


B=???
35,363.33
4.2 Price control=80%*Pe3
Pct=???
114.44
What is type of price?
Ceiling price
Result of that price?
Qd=1.5*(500-2P)=750-3P
Qs=1.4*(3P-200)=4.2P-280
Qd=???
406.67
Qs=???
200.67
Excess demand or excess supply?
Excess demand
206.00
Government spending ???
B=???
206.00 *
Pi
4.3 Tax (T)=10%Pe3
T=???
14.31
4.3.1 If taxe on the demand side (buyers)
* Demand function and supply function after tax
* Equilibrium price and quantity after tax (Pe4, Qe4)
* Incidence of tax between buyers and selllers
* Total government tax revenue is?
(D2) Q==750-3P
(S2) Q==4.2P-280
(D2Tx) Q=750-3P-3*Tx
(S2Tx) Q=4.2P-280
Pe4=
137.09
Qe4
338.72
Seller
5.96 Pe3-Pe4
Buyer
8.34 Tx-Seller
Tax revenue=
Tx*Qe4

4.3.2 If taxe on the supply side (sellers)


* Demand function and supply function after tax
(D2) Q=750-3P
(S2Tx) Q=4.2P-280 +4.2*Tx
* Equilibrium price and quantity after tax (Pe5, Qe5)
Pe5=
151.40
Qe5=
295.80
* Incidence of tax between buyers and selllers
Buyers
5.96
Sellers
8.34
* Total government tax revenue is?
4,231.56
(D) P=a+bQ
(D) Q=c+dP

(S) P=a+bQ
(S) Q=c+dP
Tx

(DTx) P=a+bQ -Tx


(DTx) Q=c+dP +dTx
d<0

Tx
(STx) P=a+bQ +Tx
(STx) Q=c+dP -dTx
d>0

Exercise 2:
Demand and Supply for commodity X:
(D1) P=
500.00 +
-0.33 Q
(S1) P=
-200.00 +
0.75 Q
1. Find the equilibrium price and quantity (Pe1, Qe1) for commodity X,
elasticity of demand and supply at equilibrium point (Ed1, Es1)?
Qe1=
646.15
Ed1=
-1.32
Pe1=
284.62
Es1=
0.59
2. Demand for commodity X increase 50% in order to become D2,
(D2) P=
500.00 +
-0.22 Q
Find the equilibrium price and quantity (Pe2, Qe2) between D2 and S1
Qe2=
720.00
Ed2=
-4.13
Pe2=
660.00
Es2=
1.22
3. Supply for commodity X increase 40% in order to become S2,
(S2) P=
-200.00 +
0.54 Q
Find the equilibrium price and quantity (Pe3, Qe3) between D2 and S2
Qe3=
923.56
Ed3=
-1.44
Pe3=
294.76
Es3=
0.60
4. From D2 and S2, the government set the price control (Pct)
4.1 Price control=120%*Pe3
Pct=???
353.72
What is type of price?
Floor price
Result of that price:
Qd=???
658.27
Qs=???
1,033.61
Excess demand or excess supply?
Excess supply
375.33
Government spending (B) ???
B=???
132,761.88

4.2 Price control=80%*Pe3


Pct=???
235.81
What is type of price?
Ceiling Price
Result of that price?
Qd=???
1,188.85
Qs=???
813.51
Excess demand or excess supply?
Excess demand
375.33
Government spending ???
B=???
375.33 *Pi
4.3 Tax (T)=10%Pe3
Tax=
T=???
29.48
4.3.1 If taxe on the demand side (buyers)
* Demand function and supply function after tax
(D2Tx) P=
470.52 +
-0.22 Q
(S2Tx) P=
-200.00 +
0.54 Q
* Equilibrium price and quantity after tax (Pe4, Qe4)
Qe4=
884.67
Ed4=
Pe4=
273.93
Es4=
* Incidence of tax between buyers and selllers
Seller=
20.83
Buyer=
8.64
* Total government tax revenue is?
26,076.92

-1.39
0.58

4.3.2 If taxe on the supply side (sellers)


* Demand function and supply function after tax
(D2Tx) P=
500.00 +
-0.22 Q
(S2Tx) P=
-170.52 +
0.54 Q
* Equilibrium price and quantity after tax (Pe5, Qe5)
Qe5=
884.67
Ed5=
Pe5=
303.41
Es5=
* Incidence of tax between buyers and selllers
Buyer=
20.83
Seller=
8.64
* Total government tax revenue is?
26,076.92

-1.54
0.64

Ecersice 3:
The price of commodity X before tax is 10$/Kg, Tax on supply side (sellers) 3$/Kg;
Ed of commodity X=-4, Es of commodity X=2. The price of commodity X after tax?
P1=
10.00
Tx=
3.00
Ed=
-4.00
Es=
2.00
Pe=
PTx=

1.00
11.00

Ecersice 4:
The price of commodity X before tax is 10$/Kg, the price of commodity X after tax is 12 $/Kg.
Ed of commodity X=-4, Es of commodity X=2. How much tax per unit of output?
P1=
10.00
P2=
12.00
Ed=
-4.00
Es=
2.00
Tx=
6.00
Pe=Tx*Es/(Es-Ed)

Exercise 1:
Complete the following table:
Units of
TFC
TVC
Output
0.00
500.00
0.00
100.00
500.00
250.00
200.00
500.00
600.00
300.00
500.00
1,000.00
400.00
500.00
1,500.00
500.00
500.00
2,100.00

TC
500.00
750.00
1,100.00
1,500.00
2,000.00
2,600.00

AFC

AVC

ATC

MC

5.00
2.50
1.67
1.25
1.00

2.50
3.00
3.33
3.75
4.20

7.50
5.50
5.00
5.00
5.20

2.50
3.50
4.00
5.00
6.00

X
3,000.00

2,500.00

2,000.00

1,500.00

1,000.00

500.00

0.00
0.00

50.00

100.00

150.00

200.00

250.00

300.00

Q
TFC

TVC

TC

350.00

400.00

450.00

500.00

Exercise 2:
Complete the following table. Assume that units of fixed input cost $10 each and that units
of variable inputs cost $20 each.
Average
Marginal
Units of Product
of Product of
Units of Units
of Variable
Variable
Variable
Fixed
Variable Output
Input AVL
Input
TFC
TVC
Input K Input
L
QL
=Q/L
MPL=Q/L
100.00
0.00
0.00
1,000.00
0.00
100.00
20.00
600.00
30.00
30.00
1,000.00
400.00
100.00
40.00
1,500.00
45.00
37.50
1,000.00
800.00
100.00
60.00
2,000.00
25.00
33.33
1,000.00
1,200.00
100.00
80.00
2,200.00
10.00
27.50
1,000.00
1,600.00
100.00
100.00
2,300.00
5.00
23.00
1,000.00
2,000.00
Average
Marginal
Units of Product
of Product of
Units of Units
of Variable
Variable
Variable
Fixed
Variable Output
Input AVL
Input
MC
AVC
Input K Input
L
QL
=Q/L
MPL=Q/L
100.00
0.00
0.00
100.00
20.00
600.00
30.00
30.00
0.67
0.67
100.00
40.00
1,500.00
45.00
37.50
0.44
0.53
100.00
60.00
2,000.00
25.00
33.33
0.80
0.60
100.00
80.00
2,200.00
10.00
27.50
2.00
0.73
100.00
100.00
2,300.00
5.00
23.00
4.00
0.87

TC
1,000.00
1,400.00
1,800.00
2,200.00
2,600.00
3,000.00

AFC

ATC

1.67
0.67
0.50
0.45
0.43

2.33
1.20
1.10
1.18
1.30

Exercise 3:
Given the total cost function TC=10000+9Q, where Q=units of output:
a. Determine the equations for TFC and TVC, and illustrate graphically the relationships among TFC, TVC, TC.
b.Determine the equation for AFC, AVC, ATC, and MC. Graphically illustrate their relationships to one another.
TFC=10000 TVC=9Q TC=TFC+TVC

X
4,000.00
3,000.00

2,000.00
1,000.00
0.00
0.00

500.00 1,000.001,500.002,000.002,500.00

Q
TFC

TVC

TC

b.Determine the equation for AFC, AVC, ATC, and MC. Graphically illustrate their relationships to one another.

X
4.00
3.00

2.00
1.00
0.00
500.00

1,000.00

1,500.00

2,000.00

Q
AFC

ATC

MC

Relationship: AFC+AVC=ATC

AVC

Exercise 4:
Given the total cost function TC=20000+4Q+0.5Q^2, where Q=units of output:
a. Determine the equations for TFC and TVC. Graph the TFC, TVC, and TC functions, and graphically show their
relationships to one another. How would you describe the behavior of TVC as output increases?
b. Determine the equations for AFC, AVC, ATC, and MC. Graph each of these functions, and graphically show
their relationships to one another.
TFC=20000 TVC= 4Q+0.5Q^2 AFC= 20000/Q AVC= 4+0.5Q ATC = 20000/Q +4+0.5Q MC=4+Q
Exercise 5:
Given the following information:
* Q=6X, where X=units of variable input and Q=units of output.
* There are 10 units of fixed input.
* Price of the fixed inputs = $10/unit.
* Price of the variable inputs = $5/unit.
Determine the corresponding equations for TFC, TVC, TC, AFC, AVC, ATC, and MC.
TFC=
100.00
TVC=
30.00 *Q
TC=
100.00 +
30.00 *Q
AFC=
100.00 /Q
AVC=
30.00
ATC=
30.00 +
100.00 /Q
MC=
30.00

Exercise 6:
The total cost function of a shirt manufacturer is TC=10+26Q-5Q^2+0.5Q^3, where TC is in hundreds of
dollars per month and Q is output in hundreds of shirts per month.
a. What is the equation for TVC?
b. What is the equation for AVC?
c. What is the equation for ATC?
d. What is the equation for MC?
e. Plot the relationships among TFC, TVC, and TC
f. Plot the relationships among AFC, AVC, AC, and MC
TVC=26Q-5Q^2+0.5Q^3
AVC= 26-5Q+0.5Q^2
ATC=10/Q +26-5Q+0.5Q^2
MC=26-Q+1.5Q^2
TFC=10=TC-TVC
AC=AFC+AVC
MC=(AC.Q)'

Exercise 7:
Given TC=2000+15Q-6Q^2+Q^3, where Q= units of output:
Complete the following table.
Q
TFC
TVC
TC
AFC
0.00
2,000.00
0.00
2,000.00
2.00
2,000.00
14.00
2,014.00
1,000.00
4.00
2,000.00
28.00
2,028.00
500.00
6.00
2,000.00
90.00
2,090.00
333.33
8.00
2,000.00
248.00
2,248.00
250.00
10.00
2,000.00
550.00
2,550.00
200.00
12.00
2,000.00 1,044.00
3,044.00
166.67
14.00
2,000.00 1,778.00
3,778.00
142.86
16.00
2,000.00 2,800.00
4,800.00
125.00
18.00
2,000.00 4,158.00
6,158.00
111.11
20.00
2,000.00 5,900.00
7,900.00
100.00
22.00
2,000.00 8,074.00
10,074.00
90.91
24.00
2,000.00 10,728.00
12,728.00
83.33
26.00
2,000.00 13,910.00
15,910.00
76.92
28.00
2,000.00 17,668.00
19,668.00
71.43
30.00
2,000.00 22,050.00
24,050.00
66.67
32.00
2,000.00 27,104.00
29,104.00
62.50
34.00
2,000.00 32,878.00
34,878.00
58.82
36.00
2,000.00 39,420.00
41,420.00
55.56
38.00
2,000.00 46,778.00
48,778.00
52.63
40.00
2,000.00 55,000.00
57,000.00
50.00

AVC

ATC

MC

7.00
7.00
15.00
31.00
55.00
87.00
127.00
175.00
231.00
295.00
367.00
447.00
535.00
631.00
735.00
847.00
967.00
1,095.00
1,231.00
1,375.00

1,007.00
507.00
348.33
281.00
255.00
253.67
269.86
300.00
342.11
395.00
457.91
530.33
611.92
702.43
801.67
909.50
1,025.82
1,150.56
1,283.63
1,425.00

3.00
15.00
51.00
111.00
195.00
303.00
435.00
591.00
771.00
975.00
1,203.00
1,455.00
1,731.00
2,031.00
2,355.00
2,703.00
3,075.00
3,471.00
3,891.00
4,335.00

Exercise 8:
Given the following cost information:
AFC for 5 units of output is $2000
AVC for 4 units of output is $850
TC rises by $1240 when the sixth unit of output is produced
ATC for 5 units of output is $2880
It costs $1000 more to produce 1 unit of output than to produce nothing.
TC for 8 units of output is $19040
TVC increases by $1535 when the seventh unit of output is produced.
AFC plus AVC for 3 units of output is $4185
ATC falls by $5100 when out rise from 1 to 2 units.
Using this information, complete the following table:
Output
TFC
TVC
TC
AFC
AVC
0.00
10,000.00
0.00
10,000.00
1.00
10,000.00 1,000.00
11,000.00
10,000.00
1,000.00
2.00
10,000.00 1,800.00
11,800.00
5,000.00
900.00
3.00
10,000.00 2,555.00
12,555.00
3,333.33
851.67
4.00
10,000.00 3,400.00
13,400.00
2,500.00
850.00
5.00
10,000.00 4,400.00
14,400.00
2,000.00
880.00
6.00
10,000.00 5,640.00
15,640.00
1,666.67
940.00
7.00
10,000.00 7,175.00
17,175.00
1,428.57
1,025.00
8.00
10,000.00 9,040.00
19,040.00
1,250.00
1,130.00

ATC

MC

11,000.00
5,900.00
4,185.00
3,350.00
2,880.00
2,606.67
2,453.57
2,380.00

1,000.00
800.00
755.00
845.00
1,000.00
1,240.00
1,535.00
1,865.00

Exercise 9:
9.1 Use the production function:
Q=10K^(0.5)*L^(0.6)
K
6.00
24.49
37.13
5.00
22.36
33.89
4.00
20.00
30.31
3.00
17.32
26.25
2.00
14.14
21.44
1.00
10.00
15.16
1.00
2.00

%Q={[1+%TC]^(alpha+beta)}-1

47.35
43.23
38.66
33.48
27.34
19.33
3.00

56.27
51.37
45.95
39.79
32.49
22.97
4.00

64.34
58.73
52.53
45.49
37.14
26.27
5.00

71.77
65.52
58.60
50.75
41.44
29.30
6.00

a. Complete the production table.


b. For this production system, are returns to scale decreasing, constant, or increasing? Explain.
c. Suppose the wage rate is $28, the price of capital also is $28 per unit, and the firm currently is
producing 30.31 units of output per period using four units of capital and two units of labor. Is
this an efficient resource combination? Explain. What would be a more efficient combination?
%TC=
700.00%
%Q=
884.92%
Q2=
98.49

9.2 Use the data from 9.1 to answer the following questions.
a. If the rate of capital input is fixed at three and if output sells for $5 per unit, determine the
total average, and marginal product functions and the marginal revenue product function for
labor in the following table.
L
0.00
1.00
2.00
3.00
4.00
5.00
6.00

TPL
0.00
17.32
26.25
33.48
39.79
45.49
50.75

APL

MPL

MRPL

17.32
13.13
11.16
9.95
9.10
8.46

17.32
8.93
7.23
6.31
5.70
5.26

86.60
44.66
36.15
31.54
28.50
26.29

b. Using data from part a, if the wage rate is $ 28 per unit, how much labor should be employed?
c. If the rate of labor input is fixed at 5 and the price of output is $5 per unit, determine the total,
average, and marginal product functions for capital and the marginal revenue product of capital
in the following table
5.00
K
0.00
1.00
2.00
3.00
4.00
5.00
6.00

TPK
0.00
26.27
37.14
45.49
52.53
58.73
64.34

APK

MPK

MRPK

26.27
18.57
15.16
13.13
11.75
10.72

26.27
10.88
8.35
7.04
6.20
5.61

131.33
54.40
41.74
35.19
31.00
28.03

d. Using the data from part c, if the price of capital is $40 per unit, how many units of capital
should be employed?
3.00

Exercise 10:
International Publishing has kept the following data on labor input and production of textbook
for each of eight production periods.
Production period
1
2
3
4
5
6
7
Labor Input
4
3
6
8
2
7
5
Output of Books
260
190
900
800
110
1500
2100
a. Use the data on labor input and total product to compute the average and marginal product for labor input rate from one
to eight. (Assume that a zero labor input would resuld in zero output)
L
QL
APL
MPL
1.00
50.00
50.00
2.00
110.00
55.00
60.00
3.00
190.00
63.33
80.00
4.00
260.00
65.00
70.00
5.00
2,100.00
420.00
1,840.00
6.00
900.00
150.00
-1,200.00
7.00
1,500.00
214.29
600.00
8.00
800.00
100.00
-700.00
b. Draw, on one set of axes the QL, APL, MPL

X
2,500.00
2,000.00
1,500.00
1,000.00

QL

500.00

APL

0.00
-500.00 0.00

5.00

-1,000.00
-1,500.00

10.00

MPL

8
1
50

Exercise 11:
K
L
QL
APL
MPL
TC=
VC=
FC=
AC=
MC=
AVC=

1.00
1.00
6.00
6.00
6.00
22,800.00
11,800.00
11,000.00
3,800.00
1,966.67

PK=
PL=
1.00
2.00
13.00
6.50
7.00
34,600.00
23,600.00
11,000.00
2,661.54
1,685.71
1,815.38

20.00
5.00
1.00
3.00
21.00
7.00
8.00
46,400.00
35,400.00
11,000.00
2,209.52
1,475.00
1,685.71

1.00
4.00
28.00
7.00
7.00
58,200.00
47,200.00
11,000.00
2,078.57
1,685.71
1,685.71

1.00
5.00
34.00
6.80
6.00
70,000.00
59,000.00
11,000.00
2,058.82
1,966.67
1,735.29

1.00
6.00
39.00
6.50
5.00
81,800.00
70,800.00
11,000.00
2,097.44
2,360.00
1,815.38

Exercise 1:
The following table shows the relationship between hours of study and final examination
grades in each of three classes for a particular student, who has a total of 15 hours to prepare
for these tests. If the objective is to maximize the average grade in the three classes, how many
hours should this student allocate to preparation for each of these classes? Explain your
approach to this problem.
Economics
Mathematics
Management
Hours
Grade
Hours
Grade
Hours
Grade
0.00
40.00
0.00
50.00
0.00
30.00
1.00
50.00
1.00
60.00
1.00
50.00
2.00
59.00
2.00
69.00
2.00
60.00
3.00
67.00
3.00
77.00
3.00
66.00
4.00
74.00
4.00
84.00
4.00
71.00
5.00
79.00
5.00
90.00
5.00
74.00
6.00
83.00
6.00
95.00
6.00
76.00
7.00
86.00
7.00
96.00
7.00
77.00
8.00
88.00
8.00
97.00
8.00
77.00
9.00
89.00
9.00
97.00
9.00
77.00
10.00
89.00
10.00
97.00
10.00
77.00

Economics
Hours
Grade
0
40
1
50
2
59
3
67
4
74
5
79
6
83
7
86
8
88
9
89
10
89
Management
Hours
Grade
0
30
1
50
2
60
3
66
4
71
5
74
6
76
7
77
8
77
9
77
10
77

Mathematics
Hours
Grade
0
50
1
60
2
69
3
77
4
84
5
90
6
95
7
96
8
97
9
97
10
97
M Eco

M Math

M Mana

10
9
8
7
5
4
3
2
1
0

10
9
8
7
6
5
1
1
0
0

20
10
6
5
3
2
1
0
0
0

Exercise 2:
Given the total cost function:

TC=1000+10Q-0.9Q^2+0.04Q^3
find the rate of output that results in minimum average variable cost.
AVC=VC/Q=10-0.9Q+0.04Q^2
AVCmin <=> Q= - (-0.9)/(2*0.04) = 11.25 => AVC min = 14.05
TC'=
10.00 +

-1.80 *Q

+
0.12 *Q^2

TC''=
-1.80 +
0.24 *Q
=
-1.56 <0
TC''=0 <=>
Q=
7.50
TC(0)<=TC(Q)
minTC=TC(0)=1000
Exercise 3:
MicroApplications Inc. is a small firm that specializes in the production and mail-order distribution
of computer programs for microcomputers. The acounting deparment has gathered the following
data on development and production costs for a typical program and the documenttation that must
accompany the program.
Development cost (fixed):
Program development
10,000.00
Manual preparation and typesetting
3,000.00
Advertising
10,000.00
Total
23,000.00
Variable costs per unit:
Blank disk
2.00
Loading cost
0.50
Postage and handling
1.25
Printing of the manual
2.75
Total
6.50
A typical program of this type, including the manual, sells for $40. Base on this information:

a. Determine the break even number of programs and the total revenue associate with this volume.
Q
0.00
100.00
200.00
300.00
400.00
500.00
600.00
700.00
800.00
900.00
1,880.60
686.57

FC
23,000.00
23,000.00
23,000.00
23,000.00
23,000.00
23,000.00
23,000.00
23,000.00
23,000.00
23,000.00
23,000.00
23,000.00

VC

AFC

0.00
650.00
1,300.00
1,950.00
2,600.00
3,250.00
3,900.00
4,550.00
5,200.00
5,850.00
12,223.88
4,462.69
MC
6.50
6.50
6.50
6.50
6.50
6.50
6.50
6.50
6.50
6.50
6.50

TR=
TC=

Qbep=

0.00 +
23,000.00 +
-23,000.00 +
686.57

AVC

230.00
115.00
76.67
57.50
46.00
38.33
32.86
28.75
25.56
12.23
33.50
TR
0.00
4,000.00
8,000.00
12,000.00
16,000.00
20,000.00
24,000.00
28,000.00
32,000.00
36,000.00
75,223.88
27,462.69
40.00 *Q
6.50 *Q
33.50 *Q

6.50
6.50
6.50
6.50
6.50
6.50
6.50
6.50
6.50
6.50
6.50
MR
40.00
40.00
40.00
40.00
40.00
40.00
40.00
40.00
40.00
40.00
40.00

TC
23,000.00
23,650.00
24,300.00
24,950.00
25,600.00
26,250.00
26,900.00
27,550.00
28,200.00
28,850.00
35,223.88
27,462.69

-23,000.00
-19,650.00
-16,300.00
-12,950.00
-9,600.00
-6,250.00
-2,900.00
450.00
3,800.00
7,150.00
40,000.00
0.00

b. MicroApplications has a minimum profit target of $40000 on each new program it develops.
Determine the unit and dollar volume of sales required to meet this goal.
=
40,000.00
Q=
1,880.60
TR=
75,223.88
c. While this program is still in the development stage, market prices for software fall by 25 percent
due to a significant increase in the number of programs being supplied to the market. Determine
the new break even unit and dollar volumes.
Qbep=
978.72
TR=
29,361.70
Pro=0<=>Qbep=FC/(P-AVC)
Pro=0<=>Trbep=FC/(1-AVC/P)=P*Qbep

A firm is considering the rental of a new copying machine. The rental terms of each of
AVC
the three
machines under consideration are given here:
Costs
0.03
Monthly
Fee Per
Copy
Machine
($)
($)
0.03
A
1,000.00
0.03
0.03
B
300.00
0.04
0.03
C
100.00
0.05
0.03
How many copies per month would the firm have to make for B to be a lower total cost machine
0.03
than C? For A to be lower cost than B?
Q
FC
VC
TC
AFC
0.00
1,000.00
0.00
100.00
1,000.00
3.00
1,003.00
10.00
200.00
1,000.00
6.00
1,006.00
5.00
300.00
1,000.00
9.00
1,009.00
3.33
400.00
1,000.00
12.00
1,012.00
2.50
500.00
1,000.00
15.00
1,015.00
2.00
600.00
1,000.00
18.00
1,018.00
1.67
TC(A)=
1,000.00 +
0.03 *Q
TC(B)=
300.00 +
0.04 *Q
TC(C)=
100.00 +
0.05 *Q
Q
TC(A)
TC(B)
TC(C)
TC(A)-TC(B)
0.00
1,000.00
300.00
100.00
700.00
10,000.00
1,300.00
700.00
600.00
600.00
20,000.00
1,600.00
1,100.00
1,100.00
500.00
30,000.00
1,900.00
1,500.00
1,600.00
400.00
45,000.00
2,350.00
2,100.00
2,350.00
250.00
50,000.00
2,500.00
2,300.00
2,600.00
200.00
60,000.00
2,800.00
2,700.00
3,100.00
100.00
70,000.00
3,100.00
3,100.00
3,600.00
0.00
80,000.00
3,400.00
3,500.00
4,100.00
-100.00
90,000.00
3,700.00
3,900.00
4,600.00
-200.00
100,000.00
4,000.00
4,300.00
5,100.00
-300.00

ATC

Exercise 4:
10.03

5.03
3.36
2.53
2.03
1.70
TC(B)-TC(C)
200.00
100.00
0.00
-100.00
-250.00
-300.00
-400.00
-500.00
-600.00
-700.00
-800.00
TC(A)-TC(C)
900.00
700.00
500.00
300.00
0.00
-100.00
-300.00
-500.00
-700.00
-900.00
-1,100.00

Exercise 5:
Suppose that the total cost equation (TC) for a monopolist is given by
TC=500+20Q^2
Let the demand equation be given by
P=400-20Q
What are the profit-maximizing price and quantity?
What are the Revenue-maximizing price and quantity?
Complete the following table.
Q
TC
TR
MC
0.00
500.00
0.00
0.00
1.00
520.00
380.00
40.00
2.00
580.00
720.00
80.00
3.00
680.00
1,020.00
120.00
4.00
820.00
1,280.00
160.00
5.00
1,000.00
1,500.00
200.00
6.00
1,220.00
1,680.00
240.00
7.00
1,480.00
1,820.00
280.00
8.00
1,780.00
1,920.00
320.00
9.00
2,120.00
1,980.00
360.00
10.00
2,500.00
2,000.00
400.00
11.00
2,920.00
1,980.00
440.00
12.00
3,380.00
1,920.00
480.00
13.00
3,880.00
1,820.00
520.00
14.00
4,420.00
1,680.00
560.00

MR
400.00
360.00
320.00
280.00
240.00
200.00
160.00
120.00
80.00
40.00
0.00
-40.00
-80.00
-120.00
-160.00

Profit
-500.00
-140.00
140.00
340.00
460.00
500.00
460.00
340.00
140.00
-140.00
-500.00
-940.00
-1,460.00
-2,060.00
-2,740.00

Exercise 6:
A firm sells in two markets and has constant marginal cost of production equal to $2 per unit,
FC=5. The demand equations for the two markets are as follows:
Market 1:
Market 2:
P1=14-2Q1
P2=10-Q2
Use third-degree price discrimination, what are the profit-maximizing price and quantities
in each market? Show that greater profits result from price discrimination than would be
obtained if a uniform price were used.
MC=
2.00
FC=
5.00
Solution 1
MR1=
14.00 +
-4.00 Q1
MR2=
10.00 +
-2.00 Q2
Price discrimination
MR1=MC<=> Q1=
3.00
P1=
8.00
TR1=
18.00
MR2=MC<=> Q2=
4.00
P2=
6.00
TR2=
16.00
=
29.00
non-price discrimination
P1=14-2Q1
3P=34-2Q
P2=20-2Q2
P=34/3-2/3Q
MR=34/3-4/3Q
Q=
TR=
TC=
=
Greater =

1.33

MC=2
7.00 P1=
46.67
19.00
27.67

6.67

Exercise 7:
An automobile manufacturer estimates that total variable costs will be $500 million and
total fixed costs will be $1 billion in the next year. In setting prices, it is assumed that
sales will be 80% of the firm's 125000 vehicle per year capacity, or 100000 units. The
target rate of return is 10 percent, which is to be earned on an investment of $2 billion.
If prices are set on a cost-plus basic, what price should be charged for each automobile?
P=AC+ Pro/Unit
VC=
500.00
FC=
1,000.00
TC=
1,500,000,000.00
Pro=
200,000,000.00
Pro/Unit=
2,000.00
AC=
15,000.00
P
17,000.00

Exercise 8:
Smith Distributing sells video cassettes in two separable markets. The marginal cost
of each cassettes is $2, FC=10. For the first market, demand is given by: Q1=20-5P1, The demand
equation for the second market is: Q2=20-2P2
a. If the firm uses third degree price dismination, what will be the profit maximizing price
and quantity in each market? How much economic profit will the firm earn?
b. If the firm charges the same price in both markets, what will be the profit maximizing
price and total quantity? How much economic profit will the firm earn?
TC=FC+AVC*Q(TC)'=MC=AVC
TC=10+2Q
Q1=20-5P1
<=>P1=(20/5)-(1/5)Q1
5P1=20-Q1
Q2=20-2P2
<=>P2=(20/2)-(1/2)Q2
2P1=20-Q2
=>MR1=(20/5)-(2/5)Q1
Q=40-7P
7P=40-Q
=>MR2=(20/2)-(2/2)Q2
P=(40/7)-(1/7)Q
=>MC=MR1=>Q1=
5.00
=>P=(40/7)-(1/7)Q
=>P1=
3.00
=>MR=(40/7)-(2/7)Q
=>TR1=
15.00
MR=MC=>
=>MC=MR2=>Q2=
8.00
Q=
13.00
=>P2=
6.00
P=
3.86
=>TR2=
48.00
TR=
50.14
TC=
36.00
TC=
36.00
TR=
63.00
Pro=
14.14
Pro=
27.00

Exercise 9:
ging bi 6, 7
Global motors sells its automobies in both the United States and Japan. Due to trade
restriction, a vehicle sold in one country cannot be resold in the other. The demand
functions for the two countries are
US
P=30000-0.4Q
Japan
P=20000-0.2Q
The firm's total cost function is TC=10,000,000+12,000Q. What price should Global
charge in each country in order to maximize profit? What will be the total profit?
MR(US)=
MR(JP)=

30,000.00 +
20,000.00 +

MC=TC'=
MR(US)=MC <=>Q=
MR(JP)=MC <=>Q=

12,000.00

TR(US)
TR(JP)
TC=
=

472,500,000.00
320,000,000.00
520,000,000.00
272,500,000.00

-0.80 *Q
-0.40 *Q

22,500.00
20,000.00

P=
P=

21,000.00
16,000.00

Exercise 10:
ly o hm
A firm produces two types of calculators, x and y. The revenue and cost equations are
shown below with Qx and Qy measured in thousands of calculators per year.
Total revenue=2Qx+3Qy
Total cost=Qx^2-2Qx*Qy-2Qy^2+6Qx+14Qy+14.72*10^6
a. To maximize profit, how many of each type of calculator should the firm produce?
b. What is the maximum profit the firm can earn?
=>Pro=TR-TC=-4Qx-11Qy-Qx^2+2QxQy+2Qy^2-14.72*10^6
=>Pro max<=Pro=0
=>Pro x=-4-2Qx+2Qy
=0
=>Pro y=-11+2Qx+4Qy
=0
n v l ngn
=>Qx=X=
0.50
500.00
=>Qy=Y=
2.50
2,500.00
=>Pro=
500.00

Exercise 11:
Miss X is considering the rental of a new telephone. The rental terms of each of the three
telephone under consideration are given here:
Costs
Monthly
Fee Per
Min
Telephone
($)
($)
A
40,000.00
1,000.00
B
20,000.00
1,200.00
C
50,000.00
1,250.00
and free of change per month for call: 50000
1. Find the TC function/month as TC=FC+AVC*Q for A, B and C?
2. Compare beetwen A and B?
3. Compare beetwen A, B and C?
TC(A)=
TC(B)=
TC(C)=

40,000.00 +
20,000.00 +
50,000.00

1,000.00 *Q
1,200.00 *Q
1,250.00 *Q

TC(A)
0.00
10.00
20.00
25.00
60.00
70.00
80.00
90.00
100.00
110.00
120.00

0.00
0.00
0.00
0.00
0.00
0.00
0.00
0.00
0.00
0.00
0.00

TC(B)
17,000.00
207,000.00
397,000.00
492,000.00
1,157,000.00
1,347,000.00
1,537,000.00
1,727,000.00
1,917,000.00
2,107,000.00
2,297,000.00

TC(C)
-20,000,000.00
-30,000,000.00
-40,000,000.00
-45,000,000.00
-80,000,000.00
-90,000,000.00
-100,000,000.00
-110,000,000.00
-120,000,000.00
-130,000,000.00
-140,000,000.00

TC(A)-TC(B)
-17,000.00
-207,000.00
-397,000.00
-492,000.00
-1,157,000.00
-1,347,000.00
-1,537,000.00
-1,727,000.00
-1,917,000.00
-2,107,000.00
-2,297,000.00

if Q<=
if Q>
TC(A)-TC(C)
20,000,000.00
30,000,000.00
40,000,000.00
45,000,000.00
80,000,000.00
90,000,000.00
100,000,000.00
110,000,000.00
120,000,000.00
130,000,000.00
140,000,000.00

40.00
40.00
TC(B)-TC(C)
20,017,000.00
30,207,000.00
40,397,000.00
45,492,000.00
81,157,000.00
91,347,000.00
101,537,000.00
111,727,000.00
121,917,000.00
132,107,000.00
142,297,000.00

Compare between A and B

0<Q<100
Q=100
Q>100

TC(A)>TC(B)
TC(A)=TC(B)
TC(A)<TC(B)

Compare between A,B and C

0<Q<=10
10<Q<=25
25<Q<=100
Q>100

TC(B)<TC(A)<=TC(C)
TC(B)<=TC(C)<TC(A)
TC(C )>TC(A )=TC(B)
TC(A)<TC(B)<TC(C)

X
300000

250000

200000

150000

100000

50000

0
0

50

100

150

200

Q
TC(A)

TC(B)

TC(C)

250

300

Exercise 12:
A Hotel have data:
Rooms:
200.00
Fixed cost ($/month)
200,000,000.00
=FC
Cost ($/room/day)
100,000.00
=AVC
Price ($/room/day)
250,000.00
=P
Month=30 days
1. Find the TC function/month as TC=FC+AVC*Q?
2. Find the TR function/month as TR=P*Q?
3. Find the function/month as =TR-TC?
4. Find the break even point (BEP) (Qbe; TRbe)?
5. Find Profit max/month with the data above?
6. Find the , if Q=150%Qbe?
7. Find Q/month, if =50*10^6? t li nhun = 50 triu tm sn lng
8. Find P/room/day, if =50*10^6 and Q=200%*Qbe
TC=
200,000,000.00 +
100,000.00 *Q
TR=
250,000.00 *Q
=
-200,000,000.00 +
150,000.00 *Q
=
1,333.33 =FC/(P-AVC)
2,666.67
=>TRbep=
333,333,333.33 =FC/(1-AVC/P)
=>Goal Seek=>
P=
250,000.00
Q=
1,333.33
2,666.67
TR=
333,333,333.33
666,666,666.67
TC=
333,333,333.33
466,666,666.67
Pro=
0.00
200,000,000.00
=>Qmax=
Pro=

6,000.00 =200*30
700,000,000.00 =TR-TC
700,000,000.00
Pro=TR-TC=(P*Q)-(FC+AVC*Q)=(P-AVC)*Q-FC
=>P=???
193,750.00

44.44%

Exercise 13:
A Firm have data:
FC($/month)=
AVC($/Q)=
P($/Q)=

20,000,000.00
4,000.00
12,000.00

Q=Number of Q/Month
1. Find the TC function/month as TC=FC+AVC*Q?
2. Find the TR function/month as TR=P*Q?
3. Find the function/month as =TR-TC?
4. Find the break even point (BEP) (Qbe; TRbe)?
5. If Q=200%*Qbe, find and /TC; /TR?
6. If /TC=25%; Find , Q, /TR?
7. Find the shutdown point of the firm in the short run?
/TR=X% <=> /TC=X%/(1+X%)
/TC=Y% <=> /TR=Y%/(1-Y%)
TC=
TR=
=
Qbe=
TRbe=
Q=
=
/TC=
Q=
=
/TR=
Shutdown point
P=

20,000,000.00 +
12,000.00 *Q
-20,000,000.00 +
2,500.00
30,000,000.00
5,000.00
20,000,000.00
25%
3,571.26
8,570,103.91
20%

4,000

4,000.00 *Q
8,000.00 *Q

Exercise 14:
A Firm have data:
FC($/month)=
20,000,000.00
AVC($/Kg)=
8,000.00
P($/Kg)=
12,000.00
Q (Kg/Month)=
20,000.00
1. Find the /month?
2. If Q(Kg/month) around 15000, 16000, 17000, 18000, 19000, 20000, 21000, 22000, 23000, 24000, 25000
and P ($/Kg) around 8000, 9000, 10000, 11000, 12000, 13000, 14000, 15000, 16000, 17000, 18000
Show the Profits of the firm?
23,000
24,000
-20,000,000
-20,000,000
TC=
180,000,000.00
3,000,000
4,000,000
TR=
240000000
26,000,000
28,000,000
Pro=
60,000,000.00
49,000,000
52,000,000
72,000,000
76,000,000
95,000,000
100,000,000
118,000,000
124,000,000
141,000,000
148,000,000
164,000,000
172,000,000
187,000,000
196,000,000
210,000,000
220,000,000
16,000
17,000
18,000
19,000
20,000
21,000
-20,000,000
-20,000,000
-20,000,000
-20,000,000
-20,000,000
-20,000,000
-4,000,000
-3,000,000
-2,000,000
-1,000,000
0
1,000,000
12,000,000
14,000,000
16,000,000
18,000,000
20,000,000
22,000,000
28,000,000
31,000,000
34,000,000
37,000,000
40,000,000
43,000,000
44,000,000
48,000,000
52,000,000
56,000,000
60,000,000
64,000,000
60,000,000
65,000,000
70,000,000
75,000,000
80,000,000
85,000,000
76,000,000
82,000,000
88,000,000
94,000,000
100,000,000
106,000,000
92,000,000
99,000,000
106,000,000
113,000,000
120,000,000
127,000,000
108,000,000
116,000,000
124,000,000
132,000,000
140,000,000
148,000,000
124,000,000
133,000,000
142,000,000
151,000,000
160,000,000
169,000,000
140,000,000
150,000,000
160,000,000
170,000,000
180,000,000
190,000,000

25,000
-20,000,000
5,000,000
30,000,000
55,000,000
80,000,000
105,000,000
130,000,000
155,000,000
180,000,000
205,000,000
230,000,000
22,000
-20,000,000
2,000,000
24,000,000
46,000,000
68,000,000
90,000,000
112,000,000
134,000,000
156,000,000
178,000,000
200,000,000

Exercise 1:
D:
X months:
i/month:
R:

T
100.00
12.00
1.00%
112.68
112.68

=PV
gi 1 s m
=Nper
=Rate
=FV
tr 1 s dng
=100*(1+1%)^12

Yt=Yo*(1+i)^(X)

chy ra 100

chy vo 112.68

Ya=Yb*(1+i)^(a-b)

Yo=PV=Present value
Yt=FV=Future value
X=Nper=The total muber of payment period in an annuity
i=Rate=The interest rate per period
Exercise 2:
D:
887.45
X months:
12.00
i/month:
1.00%
R:
1,000.00

=PV
=Nper
=Rate
=FV

Exercise 3:
D:
X months:
i/month:
R:

100.00
90.00
1.80%
500.00

=PV
=Nper
=Rate
=FV

Exercise 4:
D:
X months:
i/month:
R:

100.00
161.75
1.00%
500.00

=PV
=Nper
=Rate
=FV

Yt=Yo*(1+i)^(X)
Yo=Yt/[(1+i)^(X)]
Yo=Yt*(1+i)^(-X)

Yt=Yo*(1+i)^(X)
i=[Yt/Yo]^[1/X]-1

X=log(Yt/Yo)/log(1+i)
161.75

Exercise 5:
D:
i/month:
X month:
R:
After X months:
R:

Yt=Yo*(1+i)^(X)
Account A
Account B YtA=YoA*(1+iA)^(XA)
100.00
500.00 YtB=YoB*(1+iB)^(XB)
1.00%
0.20% YtA=K*YtB and XA=XB=X
60.00
60.00
=>X=log(K*YoB/YoA)/(Log((1+iA)/(1+iB)))
181.67
563.68
Account A=Account B
Nper
202.39
202.39
RA
RB
RA-RB
749.18
749.18
0.00

289.55
After X months:
RA
RB
RA/RB
R:
1,783.41
891.70
2.00
Cho 1 gi nh r dng Goal Seek, set cell KQ sau khi n tnh bng cng tnh, to value: hng s, by changing sell gi nh
Exercise 6:
PMT:
-100.00
X months:
60.00
i/month:
1.00%
FV=
8,166.97
Account A=2 Account B
289.55

(1 i ) X 1
FV PMT *
i
1 (1 i ) X
PV PMT *
i
PMT=The payment made each period and cannot change over the life of the annuity.
Exercise 6:
PMT:
-100.00
X months:
42.23
i/month:
0.80%
FV=
5,000.00

Exercise 7:
PMT:
-100.00
X months:
24.00
i/month:
5.88%
FV=
5,000.00
Exercise 8:
PMT:
-955.82
X months:
10.00
i/month:
1.00%
FV=
10,000.00
Exercise 9:
PMT:
-100.00
X months:
60.00
i/month:
1.00%
PV=
4,495.50
Exercise10:
PMT:
-100.00
X months:
64.11
i/month:
0.80%
PV=
5,000.00
Exercise 11:
PMT:
-100.00
X months:
24.00
i/month:
-5.19%
PV=
5,000.00

Exercise 12:
PMT:
-1,055.82
X months:
10.00
i/month:
1.00%
PV=
10,000.00
Exercise 13:
Real interest rate/
Real interest rate/
Real interest rate/
Real interest rate/
Real interest rate/
Real interest rate/
Real interest rate/
Real interest rate/
Real interest rate/
Real interest rate/
Real interest rate/
Real interest rate/
Real interest rate/
Real interest rate/
Real interest rate/
Real interest rate/
Real interest rate/
Real interest rate/
Real interest rate/
Real interest rate/
Real interest rate/
Real interest rate/
Real interest rate/
Real interest rate/

1
2
3
4
5
6
7
8
9
10
11
12
13
14
15
16
17
18
19
20
21
22
23
24

month=
months=
months=
months=
months=
months=
months=
months=
months=
months=
months=
months=
months=
months=
months=
months=
months=
months=
months=
months=
months=
months=
months=
months=

1.00%
2.01%
3.03%
4.06%
5.10%
6.15%
7.21%
8.29%
9.37%
10.46%
11.57%
12.68%
13.81%
14.95%
16.10%
17.26%
18.43%
19.61%
20.81%
22.02%
23.24%
24.47%
25.72%
26.97%

1.00%
2.00%
3.01%
4.04%
5.08%
6.12%
7.18%
8.24%
9.32%
10.41%
11.51%
12.62%
13.74%
14.87%
16.01%
17.17%
18.33%
19.51%
20.70%
21.90%
23.11%
24.34%
25.57%
26.82%

0.99%
1.98%
2.99%
4.00%
5.02%
6.06%
7.10%
8.16%
9.23%
10.30%
11.39%
12.49%
13.59%
14.71%
15.84%
16.99%
18.14%
19.30%
20.48%
21.67%
22.86%
24.07%
25.30%
26.53%

0.98%
1.96%
2.96%
3.96%
4.98%
6.00%
7.03%
8.08%
9.13%
10.20%
11.27%
12.36%
13.46%
14.56%
15.68%
16.81%
17.95%
19.10%
20.26%
21.44%
22.62%
23.82%
25.03%
26.25%

0.97%
1.94%
2.93%
3.92%
4.93%
5.94%
6.97%
8.00%
9.04%
10.10%
11.16%
12.24%
13.32%
14.42%
15.52%
16.64%
17.77%
18.91%
20.06%
21.22%
22.39%
23.57%
24.77%
25.97%

0.96%
1.92%
2.90%
3.89%
4.88%
5.89%
6.90%
7.92%
8.96%
10.00%
11.05%
12.12%
13.19%
14.27%
15.37%
16.47%
17.59%
18.72%
19.85%
21.00%
22.16%
23.33%
24.51%
25.70%

0.95%
1.91%
2.87%
3.85%
4.84%
5.83%
6.83%
7.85%
8.87%
9.90%
10.95%
12.00%
13.06%
14.14%
15.22%
16.31%
17.42%
18.53%
19.65%
20.79%
21.94%
23.09%
24.26%
25.44%

0.94%
1.89%
2.85%
3.81%
4.79%
5.78%
6.77%
7.77%
8.79%
9.81%
10.84%
11.89%
12.94%
14.00%
15.07%
16.15%
17.25%
18.35%
19.46%
20.58%
21.72%
22.86%
24.02%
25.18%

0.93%
1.87%
2.82%
3.78%
4.75%
5.72%
6.71%
7.70%
8.71%
9.72%
10.74%
11.77%
12.82%
13.87%
14.93%
16.00%
17.08%
18.17%
19.27%
20.39%
21.51%
22.64%
23.78%
24.94%

0.92%
1.86%
2.80%
3.75%
4.70%
5.67%
6.65%
7.63%
8.63%
9.63%
10.64%
11.67%
12.70%
13.74%
14.79%
15.85%
16.92%
18.00%
19.09%
20.19%
21.30%
22.42%
23.55%
24.69%

0.92%
1.84%
2.77%
3.71%
4.66%
5.62%
6.59%
7.57%
8.55%
9.54%
10.55%
11.56%
12.58%
13.61%
14.65%
15.70%
16.76%
17.83%
18.91%
20.00%
21.10%
22.21%
23.33%
24.46%

0.91%
1.82%
2.75%
3.68%
4.62%
5.57%
6.53%
7.50%
8.47%
9.46%
10.45%
11.46%
12.47%
13.49%
14.52%
15.56%
16.61%
17.67%
18.74%
19.81%
20.90%
22.00%
23.11%
24.23%

0.90%
1.81%
2.73%
3.65%
4.58%
5.53%
6.48%
7.43%
8.40%
9.38%
10.36%
11.36%
12.36%
13.37%
14.39%
15.42%
16.46%
17.51%
18.57%
19.63%
20.71%
21.80%
22.89%
24.00%

S-ar putea să vă placă și